what is the midpoint of (2,3) and (11,16)

Answers

Answer 1

Answer: Midpoint = (11.5, 14.5)

Concept:

Here, we need to know the idea of the midpoint formula.

[tex]Midpoint=(\frac{x_1+x_2}{2}, \frac{y_1+y_2}{2} )[/tex]

Solve:

Given information

(x₁. y₁) = (2, 3)

(x₂, y₂) = (11, 16)

Given expression

[tex]Midpoint=(\frac{x_1+x_2}{2}, \frac{y_1+y_2}{2} )[/tex]

Substitute values into the expression

[tex]Midpoint=(\frac{2+11}{2}, \frac{3+16}{2} )[/tex]

Simplify the numerator by addition

[tex]Midpoint=(\frac{13}{2}, \frac{19}{2} )[/tex]

Simplify by division

[tex]\boxed{Midpoint=(11.5, 14.5 )}[/tex]

Hope this helps!! :)

Please let me know if you have any questions


Related Questions

CORRECT ANSWER ONLY PLEASE!!! The table of values represents the function g(x) and the graph shows the function f(x).

Answers

Answer: Choice C.  f(x) and g(x) have a common x intercept.

Explanation:

The graph of f(x) shows it has an x intercept at x = 2, since this is one location where the curve crosses the x axis.

The g(x) table shows that x = 2 and y = 0 pair up together. This is the location (2,0). This location is the exact same x intercept as described earlier.

This is the common x intercept the two functions share. This means the two functions cross or intersect at this location.

Select the correct answer. Consider this absolute value function. f(x)= |x-5| How can function f be written as a piecewise function?

(multiple choice)

Answers

Answer:

Option A.

Step-by-step explanation:

An absolute value function can be written in two ways, as the absolute value sign does not allow negative values.

[tex]|x-5|=x-5[/tex]

or

[tex]|x-5|=-(x-5)=-x+5[/tex]

The only options with these equations are A. and D.

D. can be eliminated as the inequalities given for the piecewise function do not match f(x)

To see it visually, you can try graphing it on a calculator or internet graphing service.

What is 1,025 divided by 68

Answers

Answer:

15.07352941

Step-by-step explanation:

in short 15.07

5.
The table shows the estimated number of deer living in a forest over a five-year period. Are the data best represented by a linear, exponential, or quadratic model? Write an equation to model the data.

A. exponential; y = 89 • 0.62x
B. linear; y = 0.62x + 89
C. quadratic; y = 0.62x2 + 89
D. quadratic; y = 89x2 + 0.62

Answers

Answer:

A. exponential; y = 89 • 0.62x

Step-by-step explanation:

exponential; y = 89 • 0.62^x

Answer:

Option B exponential y = 89 · 0.62x

Step-by-step explanation:

The table shows the estimated number of deer living in a forest over a five year period.

Year            Number of deers

0                     89

1                      55

2                     34

3                     21

4                     13

Now we have to find the model representing this situation. Difference in number of deer, in the forest.

We can see there is a common ratio between each successive term r =  = 0.618

r =  = 0.618

so it can be represented by an exponential model.

Shreya has a photo that is 16 cm tall and 20 cm wide. She wants to put a frame around the photo which would have the same width all around.
Represent the area of the whole frame as a polynomial in terms of the width around the photo.
A.[tex]x^{2} + 36x + 320[/tex]

B.[tex]-4x^{2} - 72x + 320[/tex]

C.[tex]4x^{2} + 72x + 320[/tex]

D.[tex]-x^{2} - 36x - 320[/tex]

Answers

A photo is usually rectangular in shape. This means that if a frame is to be put around the photo with the same width all round, it means that if the width is "x", then the new length and width will be (l + 2x) and (w + 2x) respectively where l is length and w is width.

Option C is the correct answer

Data Given;

Height of Photo: 16 cm

Width of Photo: 20 cm

Since she wants to put a frame with same width around the photo, let's represent this width as x.

Thus;

New height = 16 + 2x

New width = 20 + 2x

Area of a rectangle is given as:

A = Length * width

Thus, in this case;

A = (16 + 2x) × (20 + 2x)

Multiplying out, we have;

A =  [tex]4x^{2} + 32x + 40x + 320[/tex]

A = 4[tex]x^{2} + 72x + 320[/tex]

Looking at the given options, Option C is the correct answer

Read more at: brainly.com/question/11289399

A simple pulley with the give radius 4 in used to lift heavy objects is positioned 10 feet above ground level. Given that the pully rotates 720 degrees determine the height yo which the object is lifted.

Answers

9514 1404 393

Answer:

  50.3 inches

Step-by-step explanation:

720 degrees is 2 full turns, or twice the circumference of the pulley. The circumference is given by ...

  C = 2πr

  C = 2π(4 in) = 8π in

Then 2 times that is ...

  height lifted = 2(8π in) = 16π in ≈ 50.3 in

The object was lifted about 50.3 inches.

Find the mean, median, and mode of 14, 15, 3, 15, 14, 14, 18, 15, 8, 16.

Answers

Answer:

mean: 13.2 (average)

median: 14.5 (Center)

mode: 14 and 15 (because data is bimodal so there are two modes

Step-by-step explanation:

Answer:

mean = 13.2, median = 15.5, mode = 18

Step-by-step explanation:

mean = 14+15+3+15+14+14+18+15+8+16/10

= 132/10 = 13.2

median =[ (n/2)th term + (n/2+1)term ]/2

here n is even so this formula is applied and before this is done, the numbers given are to be arranged in ascending or descending order

= [10/2 + (10/2+1)]/2

= [5th term + 6th term]/2

= (15+16)/2

= 31/2 = 15.5

mode = 18

Find the unknown sizes of the angle​

Answers

Answer:

b=58 ( vertically opposite angle)

58+65+x=180 (Beaing straight angle)

123+x=180

x=180-123

:.x=57

65+b+c=180 (beaing straight angle)

65+58+c=180

123+c=180

c=180-123

c=57

a=65 (beaing vertically opposite angle)

answer

:.b=57

:.c=57

:.a=65

a=65°
b=58°
c=180-65-58=57°

Hope my answer helped u :)

Rewrite each angle in radian measure as a multiple of π.

540°


Rewrite each angle in degree measure.

-4π

Answers

Answer:

Step-by-step explanation:

540° = 360° x 1 + 180°

= 2π + π

= 3π

CMIIW

What is the least positive integer by which 1922 should be divided so that the resultant integer is a
perfect square?

Answers

Answer:

1922 = 2× 31 × 31

Therefore, 1922 should be divided by 2 to get 961 which is a square of 31.

I hope this helped!

Determine the domain of the following graph:
y
12
11
9
8
7
EN WAero 5
5
4
2
-1
-12-11-10 - -8 -7 -6 -5 -4 -3 -2 -1,
-1
1
2 3 4 5 6 7 8
9 10 11 12
-2
-3
-4
-5
-6
-7
-8
-9
-10
-11

Answers

Answer:

Evaluate the expression, given functions f and h:  

f(x) = 3x − 1,

 

h(x) = −2x2 + 3x − 2.

f  

7

3

− h(−2)

Step-by-step explanation:

please answer this question​

Answers

Answer:

The answer is 0.15372

Step-by-step explanation:

A class is made up of 8 boys and 4 girls. Half of the girls wear glasses. A student is selected at random from the class. What is the probability that the student is a girl with glasses?

Answers

Answer:

1/6

Step-by-step explanation:

The total amount of people 8+4=12

there are 4/2=2 girls with glasses

the probability to chhose a girl with glasses is 2/12=1/6

3. When six is added to six times a number the result obtained is the same as when the number is added to 10 and
the result multiplied by 3. Form an equation and solve it to find the unknown number.

Answers

Answer:

Equation x(6+6)=3(10+x)

x=3.33

Step-by-step explanation:

Form an equation:

x(6+6) The result of 6+6 is multiplied by x, so you would put 6+6 in a bracket (you do what's inside the bracket first in BODMAS) and the x on the outside so you times the result by x.

3(10+x) The number x is added to 10 and the result is multiplied by 3, so the 10+x would go in a bracket with 3 on the outside.

These equal each other, so you would put an equals sign between it to make:

x(6+6)=3(10+x)

Solve to find x:

12x=30+3x You expand the brackets to make this.

9x=30 You would move 3x to the other side by doing the inverse and taking away 3x from 12x.

x=3.33 To get x you would have to do 30 divided by 9 which is 3.33.

Hope this helps :)

The number asked is 8

What is an equation?

The definition of an equation is a mathematical statement that shows that two mathematical expressions are equal.

Given that, When six is added to six times a number, the result obtained is the same as when the number is added to 10 and the result multiplied by 3.

Let the number be x,

Establishing, the equation,

6x+6 = 3(x+10)

6(x+1) = 3(x+10)

6/3(x+1) = x+10

2(x+1) = x+10

2x+2 = x+10

x = 8

Hence, the required number is 8.

For more references on equations, click;

https://brainly.com/question/29657983

#SPJ2

help please asap its timed​

Answers

Answer:

In a representation of 100, the percent change is just a number change, meaning 65 - 52, or 13 is that change. This means that the percent change from 65 to 52 is 13%  

Step-by-step explanation:

Scott needs to mail a USB drive to a friend. He uses 41-cent stamps and 8-cent stamps to pay $1.87 in postage. How many of each stamp did scott use?

Answers

Answer:

4 41cent stamps would be $1.64/ 3- 8 cent stamps would be 23 cents/ $1.64 + 23= $1.87

Step-by-step explanation:

An electrician charges $32 per home visit plus an additional $22.75 per hour. He arrives at a costumers home at 9:00am. The final bill for the visit is $128.69. At approximately what time did the electrician Lea e the costumers home?

Answers

Answer:

well since you said 32 per home since that wasnt a hour he was there, i took off 32 from the 128.69 and got 96.69 and divided that by 22.75 and got 4.2 hours but if that's not what you are talking about ill also give the answer for 128.69 divided by 22.75 and the answer for that is 5.6

Step-by-step explanation:

so just add 4.2 or 5.6 to hours and you will get you question at what time he left

(3/5) + (-6/7)+(-2/5)+(3/14) find

Answers

Answer:

[tex] \frac{ - 31}{70} [/tex]

Step-by-step explanation:

[tex] \frac{3}{5} + ( \frac{ - 6}{7} ) + ( \frac{ - 2}{5} ) + \frac{3}{14} [/tex]

[tex] = \frac{42 + ( - 60) + ( - 28) + 15}{70} [/tex]

[tex] = \frac{42 + 15 - 60 - 28}{70} [/tex]

[tex] = \frac{57 - 88}{70} [/tex]

[tex] = \frac{ - 31}{70} (ans)[/tex]

Someone help please

1= 39 degrees
2: 40 degrees
3: 41 degrees
4: 42 degrees

Answers

Answer:

A

Step-by-step explanation:

This uses the sine law.

Sin(theta)/113 = sin(42)/120             Multiply both sides by 113

sin(theta) = 113*sin(42)/120

sin42 = 0.6691

sin(theta) = 113 * 0.6691 / 120

sin(theta) = 0.630008

theta = sin-1(0.63008)

theta = 39.1    Looks Like A

How to find a domain and range and express it in a function

Answers

Answer: To find the excluded value in the domain of the function, equate the denominator to zero and solve for x .

Step-by-step explanation:

For example, in the problem below The domain of the function is set of real numbers except −3 . The range of the function is same as the domain of the inverse function. So, to find the range define the inverse of the function.

What is 10 x 0.018?

0.18
1.8
18
180

Answers

Answer:

0.18

Step-by-step explanation:

hope this helps

Answer:

0.18

Step-by-step explanation:

multiply them together

Alexa has $300 in her bank account. Each week for 5 weeks she spends 18 dollars on things. How much money does she have after 5 weeks

Answers

$210 is what she has in her bank account after 5 weeks.
18*5 =90
300-90= 210

Answer:

$210

Step-by-step explanation:

$300 - (5 x 18)

= 300 - 90

= $210

a text message plan costs $1 per month plus $0.41 per text. find the monthly cost for x text messages

the monthly cost of x messages is _ dollars

Answers

Answer:

Cost = $1 + $0.41x

Step-by-step explanation

solve for y 7y-3y=8
plzzzz answer
will give brainliest!!!!

Answers

7y - 3y = 8
4y = 8
y = 8/4
y = 2
Hope this helps :)

Answer:

Step-by-step explanation:

7y-3y= 4y

4y=8

Here when its like this we need y by itself, so you take out 4 by dividing 8 by 4.

y= 2

write the 3 terms of ( 2a+ax)^5 given the first terms in the expansion (b +2x) (2+ ax)^5 are 96 - 176x+cx^2. find the values of a,b,c

Answers

Answer:

a^5x^5, 10a^5x^4, 40a^5x^3

Step-by-step explanation:

Use pascal's triangle for the first one

(2+x)^5 * a^5

= x^5a^5 + 5*2^1*x^4*a^5 + 10*2^2*x^3*a^5 ...

= a^5x^5 + 10a^5x^4+ 40a^5x^3 ...

Pls help me :) I would appreciate it

Answers

Step-by-step explanation:

you are just using the number after the ; and switching it for the varible and seeing if the function is true or false (points :p)

H(x) = 1/x+2 and k(x) = 3x - 4
h[k(x)] =

A.(3x - 4)/(x+2)
B. 1/(4x - 2)
C. 1/(3x - 2)

Answers

Answer:

 = 1/ (3x-2)

Step-by-step explanation:

H(x) = 1/(x+2)

k(x) = 3x - 4

h[k(x)] =  place the function k(x) in for x in the function h(x)

           = 1/(3x-4 +2)

Combine like terms

           = 1/ (3x-2)

Hey there! I'm happy to help!

What we need to do is take the value of k(x) which is 3x-4 and plug it into the function h(x).

This is because our x in the function h[k(x)] is k(x), so we will substitute that value for x in that formula and that will give us our answer.

H[(k(x)]=1/(3x-4+2)

And combine like terms

H[(k(x)]=1/(3x-2)

So, our answer is C. 1/(3x-2)

Have a wonderful day and keep on learning! :D

The set S = {x\-4 Plz help ASAP

Answers

Answer:

[ - 4, 8 ]

Step-by-step explanation:

parenthesis ( ) is used when end values are not equal to x

brackets [ ] are used when the end values are equal to x

For - 4 ≤ x ≤ 8

Then x can equal both - 4 and 8 , so

[ - 4, 8 ] ← is the appropriate interval notation

cosA_1+sin1 + 1+sinA_cosA=2secA​

Answers

Step-by-step explanation:

all steps are done in the picture

what is an expression for...

4x + 4x + 4x + 4x + 4x + 4x

Answers

6(4x) would be the answer

Answer:

24x

Step-by-step explanation:

4x + 4x + 4x + 4x + 4x + 4x

=> 4(x + x + x + x + x + x)

=> 4(6x)

=> 24x